Can anyone explain why S3Q24 on PT 93 is B and not C? I know C has the conclusion less clearly delineated but other than that it seems to match up slightly better.
Help! I can't wrap my head around this one AT ALL. I'm not even sure what the conclusion is.
http://7sage.com/lsat_explanations/lsat-60-section-3-question-24/
http://7sage.com/lsat_explanations/lsat-26-section-3-question-24/
This question is beyond sucky. Did anyone else have any problems with determining the right answer for this question? Thanks.
http://7sage.com/lsat_explanations/lsat-17-section-3-question-15/
Kinda confused about this one, and why the anser is B. What does the vocal tract have to do with anything?
http://7sage.com/lsat_explanations/lsat-17-section-3-question-19/
Why is A incorrect? Is it because having the tallest peaks in the region doesn't mean it has the absolute tallest peaks?
http://7sage.com/lsat_explanations/lsat-59-section-3-question-24/
Answer is E. What does this even mean? :O
E) "presumes,without providing justification, that 18 century European aesthetics is as encompassing as an aesthetic theory can be"
https://7sage.com/lsat_explanations/lsat-46-section-3-question-24/
This sufficient assumption question really has me thrown. I've read the Manhattan explanations on this, but I'm still having a hard time with understanding the whole question. ...
This is a necessary assumption question. I correctly identified what is the premise ( aesthic value --> possible for at least two readers to agree on the interpretation) and the conclusion ( objective evaluation of a poem is possible -> popular ...
I am not sure why answer choice D in question 68 section 3 of PT 68 is incorrect. Here is an explanation for why I think D can be considered the correct answer.
I made 3 assumptions about parallel reasoning questions which I would like to ...
This was obviously a tough question, and after hours of tearing out my hair, I understand where the flaw is and why answer choice E is correct. Yet, there is still one component I am confused about.
I am quite puzzled by the answer to this question altogether. The answer to this is answer choice A (circular reasoning of the first sentence and the last part of the last sentence following "because"), but I am not quite sure why ...
I had a question on answer choice E with this one. To me, the reasoning of flaw in answer choice E was the same as that of answer choice D of the same PT same section question 20.
Can someone please confirm that I have this chain correct? I became confused with the "cannot" in the first premise. Now I'm presuming "cannot" is modifying the sufficient clause since this premise includes "unless." Please correct me if I'm wrong. I was ...
I thought that this was an example of a part to whole fallacy. The author concludes that the decrease in revenue is exaggerated because part (parts and service companies) of the industry have succeeded even after admitting that manufacturers' share of the ...
Hi everyone!
Can someone please help me with this question. I can't understand why C is wrong at all. I got it down to A and C and ended up picking C.
cant C be correct because if drinking one glass doesn't cause deficiency in vitamin D, ...
Really need some help here. I chose D and felt pretty strong about it and it turns out its B. I usually have an inkling on this but I don't see it at all. Thought it was statistical/actual issue.